MATH 201 Recitation Problems

Recitation 14 week of Dec 12, 2022

Final exam review

Recitation 13 week of Dec 5, 2022

Question 1:

Let $Y$ be geometric random variable with parameter $p=1/6$.

(a) Use Markov’s inequality to find an upper bound for ${\rm{P}}(Y\geq 16)$.

(b) Use Chebyshev’s inequality to find an upper bound for ${\rm{P}}(Y\geq 16)$.

(c) Explicitly compute the probability ${\rm{P}}(Y\geq 16)$ and compare it with the upper bounds you derived.

Solution

(a) Since $Y \sim \text{Geom}(\frac{1}{6})$, ${\rm{E}}[Y]=6$, moreover geometric random variables take nonnegative values, hence we can apply Markov’s inequality to get

\[\begin{align*} {\rm{P}}(Y\geq 16) \leq \frac{{\rm{E}}(Y)}{16}=\frac{6}{16} =0.375. \end{align*}\]

(b) Since $Y \sim \text{Geom}(\frac{1}{6})$, ${\rm{Var}}(Y)=\frac{1-1/6}{(1/6)^2}=30$, and applying Chebyshev’s inequality, we get

\[\begin{align*} {\rm{P}}(Y\geq 16) ={\rm{P}}(Y-6\geq 10) \leq {\rm{P}}(|Y-6|\geq 10) \leq \frac{{\rm{Var}}(Y)}{(10)^2}=\frac{30}{100}=0.3. \end{align*}\]

(c) Since $Y \sim \text{Geom}(\frac{1}{6})$,

\[{\rm{P}}(Y \geq 16)={\rm{P}}(Y > 15)= \left(\frac{5}{6}\right)^{15} \approx 0.065.\]
Question 2:

Suppose that $X$ is a nonnegative random variable with ${\rm{E}}[X]=10$.

(a) Give an upper bound on the probability that $X$ is larger than 15.

(b) Suppose that we also know that ${\rm{Var}}(X)=3$. Give a better upper bound on ${\rm{P}}(X>15)$ than in part (a).

(c) Suppose that $Y_1,\dots, Y_{300} $ are i.i.d random variables with the same distribution as $X$ so that in particular ${\rm{E}}[Y_i]=10$ and ${\rm{Var}}(Y_i)=3$. Estimate the probability that $\sum_{i=1}^{300} Y_i$ is larger than 3030.

Solution (a) Since ${\rm{E}}[X]=10$, moreover $X$ take nonnegative values, hence we can apply Markov’s inequality to get \(\begin{align*} {\rm{P}}(X\geq 15) \leq \frac{{\rm{E}}(X)}{15}=\frac{10}{15} \approx 0.666. \end{align*}\)

(b) Since ${\rm{Var}}(X)=3$, and applying Chebyshev’s inequality, we get \(\begin{align*} {\rm{P}}(X\geq 15) ={\rm{P}}(X-10\geq 5) \leq {\rm{P}}(|X-10|\geq 5) \leq \frac{{\rm{Var}}(X)}{5^2}=\frac{3}{25}=0.12. \end{align*}\)

(c) Let $S_n=\sum_{k=1}^n Y_i$. Since $Y_i$’s are iid with finite mean and variance, we can apply the CLT to get \(\begin{align*} \rm{P}(S_{300}\geq 3030) &=\rm{P}\left(\frac{S_{300}-(300)(10)}{\sqrt{(300)(3)}}\geq \frac{3030-(300)(10)}{\sqrt{(300)(3)}} \right)\\&=\rm{P}\left(\frac{S_{300}-3000}{30}\geq 1 \right)\approx 1-\Phi(1)=1 - 0.8413=0.1587 \end{align*}\)

Question 3:

Every morning I take either bus number 5 or bus number 8 to work. Every morning the waiting time for the number 5 is exponential with mean 10 minutes, while the waiting time for the number 8 is exponential with mean 20 minutes. Assume all waiting times are independent of each other. Let $S_n$ denote the total amount of bus-waiting (in minutes) that I have done during $n$ mornings, and let $T_n$ be the number of times I have taken the number 5 bus during $n$ mornings.

(a) Find the limit $\lim_{n\to \infty} {{\rm{P}}(S_n\leq 7n)}$.

(a) Find the limit $\lim_{n\to \infty} {{\rm{P}}(T_n\geq 0.6n)}$.

Solution

Let $X_i$ be waiting time for the bus 5 on the $i$-th day and $Y_i$ waiting time for the bus 8 on the $i$-th day. Then we are given that $X_i \sim \text{Exp}(\frac{1}{10})$ and $Y_i \sim \text{Exp}(\frac{1}{20})$. Now let us define the random variable \(\begin{align*} J_i:=\min(X_i,Y_i)\hskip 5pt \text{and }\hskip 5pt I_i:=I_{(X_i<Y_i)}. \end{align*}\)

Then we have $S_n=\sum_{i=1}^nJ_i$ and $T_n=\sum_{i=1}^n I_i$. Recall from Chapter 6 (exercises 6.33 and 6.34), we know $J_i \sim\text{Exp}(\frac{1}{10}+\frac{1}{20})=\text{Exp}(\frac{3}{20})$ and $I\sim \text{Ber}(p)$ where $p={\rm{P}}(X_i<Y_i)=\frac{1/10}{3/20}=\frac{2}{3}. $

(a) From Law of large numbers we know for any $\epsilon>0$

\[\begin{align*} { \rm{P}}\left(\left|\frac{S_n}{n}-\mu\right|> \epsilon\right) \to 0 \text{ as }n\to \infty, \end{align*}\]

and \(\begin{align*} { \rm{P}}\left(\left|\frac{S_n}{n}-\mu\right|\leq \epsilon\right) \to 1 \text{ as }n\to \infty. \end{align*}\)

Now, take $\epsilon=7-\frac{20}{3}=\frac{1}{3}$, then

\[\begin{align*} {\rm{P}}\left({S_n} \leq 7n\right)&= {\rm{P}}\left(\frac{S_n}{n} \leq 7\right)={\rm{P}}\left(\frac{S_n}{n}-\frac{20}{3} \leq 7-\frac{20}{3}\right)\geq {\rm{P}}\left(-\frac{1}{3}\leq \frac{S_n}{n}-\frac{20}{3} \leq \frac{1}{3} \right)\\&={\rm{P}}\left(\left| \frac{S_n}{n}-\frac{20}{3} \right|\leq \frac{1}{3} \right) \to 1 \text{ as } n\to\infty. \end{align*}\]

Hence

\[\begin{align*} \lim_{n\to \infty} {\rm{P}}\left(S_n \leq 7n\right)=1. \end{align*}\]

(b) Take $\epsilon=-0.6+\frac{2}{3}=\frac{1}{15}$, then

\[\begin{align*} {\rm{P}}\left({T_n} \geq 0.6n\right)&= {\rm{P}}\left(\frac{T_n}{n} \geq 0.6\right)={\rm{P}}\left(\frac{T_n}{n}-\frac{2}{3} \geq 0.6-\frac{2}{3}\right)={\rm{P}}\left(\frac{T_n}{n}-\frac{2}{3} \geq -\frac{1}{15}\right)\\&\geq {\rm{P}}\left(-\frac{1}{15}\leq \frac{T_n}{n}-\frac{2}{3} \leq \frac{1}{15} \right)={\rm{P}}\left(\left| \frac{T_n}{n}-\frac{2}{3} \right|\leq \frac{1}{15} \right) \to 1 \text{ as } n\to\infty. \end{align*}\]

Hence

\[\begin{align*} \lim_{n\to \infty} {\rm{P}}\left(T_n \geq 0.6n\right)=1. \end{align*}\]

Recitation 12 week of Nov 28, 2022

Question 1:

Suppose $X$ has moment generating function \(M_X(t) = \frac{1}{2} + \frac{1}{3}e^{-4t} + \frac{1}{6} e^{5t}.\)

(a) Find the mean and variance of $X$ by differentiating the moment generating function to find moments.

(b) Find the probability mass function of $X$. Use the probability mass function to check your answer for part (a).

Solution

(a) Note that $M’_X(t)=-\frac{4}{3}e^{-4t} + \frac{5}{6} e^{5t}$ and $M’‘_X(t)=\frac{16}{3}e^{-4t} + \frac{25}{6} e^{5t}$ and hence

\[\begin{align*} {\rm{E}}[X]&=M'_X(0)=-\frac{4}{3} + \frac{5}{6} =-\frac{3}{6}=-\frac{1}{2}\\ {\rm{E}}[X^2]&=M''_X(0)=\frac{16}{3} + \frac{25}{6} =\frac{57}{6}\\ {\rm{Var}}[X]&= {\rm{E}}[X^2]-({\rm{E}}[X])^2=\frac{57}{6}- \frac{1}{4} =\frac{111}{12} \end{align*}\]

(b) The possible values of $X$ are ${-4,0,5}$ and

\(\begin{align*} {\rm{P}}(X=-4)=\frac{1}{3}, \hskip 5pt {\rm{P}}(X=0)=\frac{1}{2}, \hskip 5pt {\rm{P}}(X=5)=\frac{1}{6}. \end{align*}\) Hence

\[\begin{align*} {\rm{E}}[X]&=-4\frac{1}{3} +0\frac{1}{2}+5 \frac{1}{6} =-\frac{3}{6}=-\frac{1}{2}\\ {\rm{E}}[X^2]&=M''_X(0)=16\frac{1}{3} +0\frac{1}{2}+ 25\frac{1}{6} =\frac{57}{6}\\ {\rm{Var}}[X]&= {\rm{E}}[X^2]-({\rm{E}}[X])^2=\frac{57}{6}- \frac{1}{4} =\frac{111}{12}. \end{align*}\]
Question 2:

The random variable $X$ has the following probability density function: \(f_X(x) = \begin{cases} xe^{-x}, \quad &\text{if } x > 0\\ 0, \quad &\text{otherwise}. \end{cases}\)

(a) Find the moment generating function of $X$.

(b) Using the moment generating function of $X$, find $\E[X^n]$ for all positive integers $n$. Your final answer should be an expression that depends only on $n$.

Solution

(a)

\[\begin{align*} M_X(t)={\rm{E}}[e^{tX}]&=\int_{-\infty}^{\infty}e^{tx} f_X(x)dx=\int_0^{\infty} e^{tx} xe^{-x}dx\\ &=\int_0^{\infty} xe^{-(1-t)x} dx=\frac{-e^{-(1-t) x} (1 + (1-t) x))}{(1-t)^2}\Big|_{x=0}^{x=\infty}\\ & =\begin{cases} \frac{1}{(1-t)^2} &\text{if }t<1\\ \infty&\text{if }t\geq 1 \end{cases} \end{align*}\]

(b) Note that for $t<1$,

\[\begin{align*} M'_X(t)=2(1-t)^{-3}, \hskip 5pt M''_X(t)=(3)(2)(1-t)^{-4}, \hskip 5pt M'''_X(t)=(4)(3)(2)(1-t)^{-4} \end{align*}\]

and in general for $t<1$

\(\begin{align*} M^{(n)}_X(t)=(n+1)!(1-t)^{-(n+2)}. \end{align*}\) So, we see

\(\begin{align*} {\rm{E}}[X]=M'_X(0)=2(1)^{-3}=2, \hskip 5pt {\rm{E}}[X^2]=M''_X(0)=(3)(2)=6, \hskip 5pt {\rm{E}}[X^3]=M'''_X(0)=(4)(3)(2)=24 \end{align*}\) and in general for $n\in {\mathbb{N}}$

\[\begin{align*} {\rm{E}}[X^n]= M^{(n)}_X(0)=(n+1)!. \end{align*}\]
Question 3:

A fair coin is flipped 30 times. Let $X$ denote the number of heads among the first 20 coin flips and $Y$ denote the number of heads among the last 20 coin flips. Compute the correlation coefficient of $X$ and $Y$.

Solution

Define for $i\in {1,2,\dots,29,30}$

\(\begin{align*} I_i=\begin{cases} 1, &\text{if ith flip is heads},\\ 0, &\text{if ith flip is tails}. \end{cases} \end{align*}\) Then $X=I_1+\dots+I_{20}$ and $Y=I_{11}+\dots+I_{30}$ where $I_i \sim \text{Ber}(\frac{1}{2})$ and all flips are independent. So we have ${\rm{E}}[I_i]={\rm{E}}[I_1]=\frac{1}{2}$, ${\rm{E}}[I_i^2]={\rm{E}}[I_1^2]=\frac{1}{2}$, ${\rm{Var}}(I_i)={\rm{Var}}(I_1)=\frac{1}{4}$, and for any $i\neq j $

\[\begin{align*} {\rm{Cov}}(I_i,I_j)&= {\rm{Cov}}(I_1,I_2)={\rm{E}}[I_1I_2]-{\rm{E}}[I_1]{\rm{E}}[I_2]\\&= {\rm{P}}(\text{first two flips are heads})- {\rm{P}}(\text{first flip is heads}) {\rm{P}}(\text{second flip is heads}) \\ &=\frac{1}{4}-\frac{1}{4}=0. \end{align*}\]

Using the linearity of the covariance in each component, we see

\[\begin{align*} {\rm{Cov}}(X,Y)&={\rm{Cov}}(I_1+\dots+I_{20},\,I_{11}+\dots+I_{30})=\sum_{i=1}^{20}\sum_{j=11}^{30} {\rm{Cov}}(I_i,I_j) \\&= \sum_{i=1}^{20}\sum_{j=11, i\neq j}^{30} {\rm{Cov}}(I_i,I_j)+\sum_{k=11}^{20} {\rm{Var}}(I_k) =\left((20)(20)-10\right){\rm{Cov}}(I_1,I_2)+(10){\rm{Var}}(I_1)\\ &=10(\frac{1}{4})=\frac{5}{2}. \end{align*}\]

Using the independence we have

\(\begin{align*} {\rm{Var}}(X)={\rm{Var}}(I_1+\dots+I_{20})={\rm{Var}}(I_1)\dots{\rm{Var}}(I_{20})=\frac{20}{4}=5. \end{align*}\) Similarly,

\(\begin{align*} {\rm{Var}}(Y)={\rm{Var}}(I_{11}+\dots+I_{30})={\rm{Var}}(I_{11})\dots{\rm{Var}}(I_{30})=\frac{20}{4}=5. \end{align*}\) Finally,

\[\begin{align*} {\rm{Corr}}(X,Y)&=\frac{ {\rm{Cov}}(X,Y)}{\sqrt{ {\rm{Var}}(X) {\rm{Var}}(Y)}} =\frac{\frac{5}{2}}{\sqrt{25}}=\frac{1}{2}. \end{align*}\]

Recitation 11 week of Nov 14, 2022

Midterm 2 Review.

Recitation 10 week of Nov 7, 2022

Question 1:

Let \(X \sim \mathrm{Poisson}(\lambda)\). Compute \({\rm{E}}\left[\frac{1}{1+X}\right]\).

Solution Recall the probability mass function of the Poisson distribution:

\(\begin{align*} {\rm{P}}(X=k)=p(k)=e^{-\lambda} \frac{\lambda^k}{k!} \text{ for }k=0,1,2,\dots. \end{align*}\) Then

\[\begin{align*} {\rm{E}}\left[\frac{1}{1+X}\right]=\sum_{k=0}^{\infty} \frac{1}{1+k}p(k)=e^{-\lambda} \sum_{k=0}^{\infty} \frac{\lambda^{k}}{(1+k)k!} =\frac{e^{-\lambda}}{\lambda} \sum_{k=0}^{\infty} \frac{\lambda^{k+1}}{(1+k)k!}=\frac{e^{-\lambda}}{\lambda} e^{\lambda}=\frac{1}{\lambda}. \end{align*}\]

To compute this sum we used the Taylor series of exponential function, and integrate both sides to get

\[\begin{align*} e^{x}=\sum_{k=0}^{\infty} \frac{x^k}{k!} \implies e^{x}=\sum_{k=0}^{\infty} \frac{x^{k+1}}{(k+1)k!} \end{align*}\]
Question 2:

On the first 300 pages of a book, you notice that there are, on average, 6 typos per page. What is the probability that there will be at least 4 typos on page 301?

Solution

For the number of times an event occur on a given interval, it is appropriate use Poisson distribution with mean $\lambda=6$ as the probability of an error is relatively low. Then

\[\begin{align*} {\rm{P}}(X \geq 4)=\sum_{k=4}^{\infty} e^{-6}\frac{6^k}{k!}=1-\sum_{k=0}^{3} e^{-6}\frac{6^k}{k!} =1-e^{-6}-\frac{6 e^{-6}}{1!}-\frac{6^2 e^{-6}}{2!}-\frac{6^3 e^{-6}}{3!}=\approx 0.849 \end{align*}\]
Question 3:

Let the random variables $X, Y$ have joint density function

\[f(x, y) = \begin{cases} 3(2-x)y \quad \text{ if } 0<y<1 \quad \text{ and } y<x<2-y, \\ 0 \quad \text{ otherwise } \end{cases}\]

Find the marginal density functions $f_X$ and $f_Y$.

Solution

Before we do any calculation first we determine the region where $f(x,y)$ is non-zero. If we plot the lines $y=x$ and $y=2-x$, we see that they intersect at $(1,1)$. The constrains $0<y<1$ and $y<x<2-y$ tell us that the region is the triangle with vertices $(0,0), (1,1), (2,0)$.

First we find $f_Y(y)$ by integrating against $x$. For a fixed value of $y \in [0,1]$, the region is bounded by $x=y$ and $x=2-y$, so

\[\begin{align*} f_Y(y) &= \int_y^{2-y} 3(2-x)y \, dx\\ &= [6xy - \frac{3}{2}x^2y]_y^{2-y}\\ &= (6(2-y)y - \frac{3}{2}(2-y)^2y) -(6y^2-\frac{3}{2}y^3)\\ &= 6y-6y^2. \end{align*}\]

so that

\[f_Y(y) = \begin{cases} 6y-6y^2 \quad \text{ if } 0\leq y \leq 1, \\ 0 \quad \text{ otherwise } \end{cases}\]

To find the marginal of $X$ we integrate $f$ against $y$. There are three cases.

Case (1): when $0\leq x \leq 1$, we integrate

\[\begin{align*} f_X(x) &= \int_0^x 3(2-x)y \, dy\\ &= [3(2-x) y^2/2]_0^x = \frac{3}{2}(2-x)x^2 \end{align*}\]

Case (2): when $1 < x \leq 2$, we integrate

\[\begin{align*} f_X(x) &= \int_0^{2-x} 3(2-x)y \, dy\\ &= [3(2-x) y^2/2]_0^{2-x} = \frac{3}{2}(2-x)^3. \end{align*}\]

The last case corresponds to $f_X(x) = 0 $ for $x$ outside of $[0,2]$, so that

\[f_X(x) = \begin{cases} \frac{3}{2}(2-x)x^2 \quad \text{ if } 0\leq x \leq 1, \\ \frac{3}{2}(2-x)^3 \quad \text{ if } 1 < x \leq 2\\ 0 \quad \text{ otherwise } \end{cases}\]
Question 4:

The joint probability mass function of $(X,Y)$ is given by the following table:

X\Y 0 1 2 3
1 1/15 1/15 2/15 1/15
2 1/10 1/10 1/5 1/10
3 1/30 1/30 0 1/10

(a) Find the marginal probability mass functions of $X$ and $Y$.

(b) Compute \({\rm{P}}(X+Y^2\leq 2)\)

Solution

(a) Probability mass function of $X$ can be obtained by summing the rows: \(\begin{align*} {\rm{P}}(X=1)&=\sum_{l=0}^3p_{(X,Y)}(1,l)=p_{(X,Y)}(1,0)+p_{(X,Y)}(1,1)+p_{(X,Y)}(1,2)+p_{(X,Y)}(1,3)\\&=\frac{1}{15}+\frac{1}{15}+\frac{2}{15}+\frac{1}{15}=\frac{1}{3}\\ {\rm{P}}(X=2)&=\sum_{l=0}^3p_{(X,Y)}(2,l)=p_{(X,Y)}(2,0)+p_{(X,Y)}(2,1)+p_{(X,Y)}(2,2)+p_{(X,Y)}(2,3)\\&=\frac{1}{10}+\frac{1}{10}+\frac{1}{5}+\frac{1}{10}=\frac{1}{2}\\ {\rm{P}}(X=3)&=\sum_{l=0}^3p_{(X,Y)}(3,l)=p_{(X,Y)}(3,0)+p_{(X,Y)}(3,1)+p_{(X,Y)}(3,2)+p_{(X,Y)}(3,3)\\&=\frac{1}{30}+\frac{1}{30}+0+\frac{1}{10}=\frac{1}{6}. \end{align*}\)

Similarly,

Probability mass function of $Y$ can be obtained by summing the columns:

\[\begin{align*} {\rm{P}}(Y=0)&=\sum_{k=1}^3p_{(X,Y)}(k,0)=p_{(X,Y)}(1,0)+p_{(X,Y)}(2,0)+p_{(X,Y)}(3,0)\\&=\frac{1}{15}+\frac{1}{10}+\frac{1}{30}+0=\frac{1}{5}\\ {\rm{P}}(Y=1)&=\sum_{k=1}^3p_{(X,Y)}(k,1)=p_{(X,Y)}(1,1)+p_{(X,Y)}(2,1)+p_{(X,Y)}(3,1)\\&=0+\frac{1}{15}+\frac{1}{10}+\frac{1}{30}=\frac{1}{5}\\ {\rm{P}}(Y=2)&=\sum_{k=1}^3p_{(X,Y)}(k,2)=p_{(X,Y)}(1,2)+p_{(X,Y)}(2,2)+p_{(X,Y)}(3,2)\\&=\frac{2}{15}+\frac{1}{5}+0=\frac{1}{3}\\ {\rm{P}}(Y=3)&=\sum_{k=1}^3p_{(X,Y)}(k,3)=p_{(X,Y)}(1,3)+p_{(X,Y)}(2,3)+p_{(X,Y)}(3,3)\\&=\frac{1}{15}+\frac{1}{10}+\frac{1}{10}=\frac{4}{15}. \end{align*}\]

(b) The pairs $(l,k)$ satisfying $l+k^2\leq 2$ are $(1,0),(1,1),(2,0)$. So,

\[\begin{align*} {\rm{P}}(X+Y^2 \leq 2)=p_{X,Y}(1,0)+p_{X,Y}(1,1)+p_{X,Y}(2,0)=\frac{1}{15}+\frac{1}{15}+\frac{1}{10}=\frac{7}{30} \end{align*}\]

Recitation 9 week of Oct 31, 2022

Question 1:

Approximate the probability that out of 300 die rolls we get exactly 100 numbers that are multiples of 3.

Solution

Multiple of 3s are $3,6$ which have probability $\frac{1}{3}$ Let $S\sim \text{Bin}(300,\frac{1}{300})$. Then ${\rm{E}}[S]=300\frac{1}{3}=100$ and ${\rm{Var}}[S]=300\frac{1}{3}\frac{2}{3}=\frac{200}{3}$. Using normal approximation with continuity correction, we see

\[{ \rm{P}}(S=100)={ \rm{P}}(99.5\leq S\leq 100.5)={ \rm{P}}\left(\frac{99.5-100}{\sqrt{(200/3)}}\leq \frac{S-100}{\sqrt{(200/3)}}\leq\frac{100.5-100}{\sqrt{(200/3)}}\right)\] \[= { \rm{P}}\left(\frac{-0.5}{100\sqrt{2}}\leq \frac{S-100}{\sqrt{200/3}}\leq\frac{0.5}{\sqrt{200/3}}\right) \approx \Phi(\frac{0.5}{\sqrt{200/3}})-\Phi(-\frac{0.5}{\sqrt{200/3}}) =2\Phi(\frac{0.5}{\sqrt{200/3}})-1\] \[\approx 2\Phi(0.06)-1\approx 2(0.5239)-1=0.0478.\]
Question 2:

81 randomly chosen individuals were interviewed to estimate the unknown fraction $p\in(0,1)$ of the population that prefers cereals to bagels for breakfast. The resulting estimate is $\hat{p}$. With what level of confidence can we state that the true $p$ lies in the interval $(\hat{p}-0.05, \hat{p}+0.05)$?

Solution

Recall

\[{\rm{P}}(|p-\hat{p}| <\epsilon) \geq 2\Phi(2\epsilon \sqrt{n})-1\]

In the set up of the question $n=81$, $\epsilon = 0.05$. Hence the confidence level is \(2\Phi(2(0.05) \sqrt{81})-1=2\Phi(0.9)-1\approx 2(0.8159)-1=0.6318.\)

Question 3:

In a game you win 10 dollars with probability $\frac{1}{20}$ and lose 1 dollar with probability $\frac{19}{20}$. Approximate the probability that you lost less than $100 after the first 200 games. How will this probability change after 300 games?

Solution

Let $S_n$ denote the number of rounds you win and $X_n$ denote the total earnings. Then $S_n \sim \text{Bin}(n,\frac{1}{20})$ and $X_n=10S_n-(n-S_n)=11S_n-n$. First we would like to estimate ${\rm{P}}(X_{200}>-100)$. We will use normal approximation for $S_{200}$. Note that ${\rm{E}}[S_{200}]=200\frac{1}{20}=10$ and ${\rm{Var}}[S_{200}]=200\frac{1}{20}\frac{19}{20}=\frac{19}{2}$. Then, with continuity correction

\[{\rm{P}}(X_{200}>-100)={\rm{P}}(11S_{200}-200>-100)= {\rm{P}}(S_{200}\geq 10)={\rm{P}}(S_{200}\geq 9.5)\] \[={\rm{P}}(\frac{S_{200}-10}{\sqrt{19/2}}\geq-\frac{0.5}{\sqrt{19/2}})\approx 1-\Phi(\frac{100-110}{11\sqrt{19/2}})\] \[\approx 1-\Phi(-0.16)=1-(1-\Phi(0.16)) =\Phi(0.16)\approx 0.563559\]

The actual probability using a computer \({\rm{P}}(S_{200} \geq 10)=1-\sum_{i=0}^{9} \binom{200}{i}\left(\frac{1}{20}\right)^i\left(\frac{19}{20}\right)^{200-i} \approx 0.54529\)

Similarly, \({\rm{P}}(S_{300}\geq 10) \approx 0.1762\)

Recitation 8 week of Oct 24, 2022

Question 1:

Suppose that $X$ is a random variable with mean 2 and variance 3.

(a) Compute $\mathrm{Var}(3X+4)$.

(b) Compute $\mathrm{E}[3X+4]$.

(c) Compute $\mathrm{E}[(3X+4)^2]$.

Solution

Note that for any $a,b$, \(\mathrm{E}[aX+b]=a \mathrm{E}[X]+b \text{ and }\mathrm{Var}(aX+b)=a^2 \mathrm{Var}(X)\)

(a) Hence

\[\mathrm{Var}(3X+4)=3^2 \mathrm{Var}(X)=27.\]

(b)

\[\mathrm{E}[3X+4]=3 \mathrm{E}[X]+4=10.\]

(c) Using $\mathrm{Var}(3X+4)= \mathrm{E}[(3X+4)^2]- \left(\mathrm{E}[3X+4]\right)^2$ and part a and b, we get

\[\mathrm{E}[(3X+4)^2]= \mathrm{Var}(3X+4)+\left(\mathrm{E}[3X+4]\right)^2=27+100=127.\]
Question 2:

Erin played a game in the casino. The dealer flips a coin until the first head shows up. If the head shows up at the $k$-th round, Erin will get $3k+1$ dollars. Assume that $\mathrm{P}(H)=p$. Find the expected payout.

Solution

Let $X$ be the the number of the flips until the first head. Then we have $X\sim \text{Geo}(p)$. And let $Y=g(X)=3X+1$ which corresponds to the earnings of Erin. Then, we have

\[\mathrm{E}[Y]= \mathrm{E}[g(X)]=\sum_{k=1}^{\infty} g(k)\mathrm{P}(X=k)=\sum_{k=1}^{\infty} (3k+1)(1-p)^{k-1}p\]

\(=3 p\sum_{k=1}^{\infty} k(1-p)^{k-1}+p\sum_{k=1}^{\infty} (1-p)^{k-1} = \frac{3p}{(1-(1-p))^2}+\frac{p}{1-(1-p)}=\frac{3}{p}+1\) where we have used the geometric sum

\[\sum_{i=0}^{\infty}x^i=\frac{1}{1-x}\]

and its derivative

\[\sum_{i=1}^{\infty}i x^{i-1}=\frac{1}{(1-x)^2}\]

with $x=1-p$.

Question 3:

Let $X$ be a normal random variable with mean 3 and variance 4.

(a) Find the probability ${\rm{P}}(2<X<6)$.

(b) Find the value $c$ such that ${\rm{P}}(X>c)=0.33$.

(c) Find ${\rm{E}[X^2]}$.

Solution

Note that $X\sim \mathcal{N}(3,4)$ and $Z=\frac{X-3}{2} \sim \mathcal{N}(0,1)$. Let $\Phi$ be the cdf of $Z$.

(a) \({\rm{P}}(2<X<6)= {\rm{P}}\left(\frac{2-3}{2}<\frac{X-3}{2}<\frac{6-3}{2}\right) ={\rm{P}}\left(-0.5<\frac{X-3}{2}<1.5\right)=\Phi(1.5)-\Phi(-0.5)\)

\(=\Phi(1.5)-(1-\Phi(0.5)) =0.9332-1+0.6915= 0.6247\) where we used $\Phi(-x)=1-\Phi(x)$ for $x>0$ and a table for $\Phi$.

(b) Note that

\[{\rm{P}}(X>c)={\rm{P}}(\frac{X-3}{2}>\frac{c-3}{2})={\rm{P}}(Z>\frac{c-3}{2}) =1-\Phi(\frac{c-3}{2})\]

So we want to solve $1-\Phi(\frac{c-3}{2})=0.33$ or $\Phi(\frac{c-3}{2})=0.67$. From the table $\frac{c-3}{2}=0.44$ is a good approximation and hence $c=3.88.$

(c) \(\mathrm{E}[X^2]=\mathrm{Var}(X)+\left(\mathrm{E}(X)\right)^2=4+3^2=13.\)

Recitation 7 week of Oct 17, 2022

Question 1:

Let $X$ be a random variable with density function \(f(x) = \begin{cases} \frac{1}{4} & 1<x<2 \\ c & 3<x<5 \\ 0 & \text{otherwise,} \end{cases}\) where $c$ is a constant.

(a) What is the value of $c$?

(b) Find $\rm{P}(\frac{3}{2} < X < 4)$.

(c) Find $\rm{E} [8X+3]$.

Solution

(a) In order $f$ to be a pdf, we must have \(1= \int_{-\infty}^{\infty}f(x)dx=\int_1^2 \frac{1}{4}\,dx+\int_3^5 c\,dx=\frac{1}{4}+2c,\) Hence $2c=\frac{3}{4}$ and $c=\frac{3}{8}$.

(b) \({\rm{P}}(\frac{3}{2}<X<4)= \int_{\frac{3}{2}}^{4}f(x)dx=\int_{\frac{3}{2}}^2 \frac{1}{4}\,dx+\int_3^4 \frac{3}{8}\,dx=\frac{1}{8}+\frac{3}{8}=\frac{1}{2}.\)

(c) First, let us compute ${\rm{E}}[X]$: \({\rm{E}}[X]=\int_{-\infty}^{\infty} xf(x)\,dx= \int_{1}^2 \frac{x}{2}\,dx+\int_3^5 \frac{3x}{8}\,dx= \frac{3}{4}+\frac{3(16)}{16}=\frac{15}{4}.\)

Now, using ${\rm{E}[X]}$ and linearity of the expectation, we get \({\rm{E}}[8X+3]= 8{\rm{E}}[X]+3=8\frac{15}{4}+3=33.\)

Question 2:

There are 6 balls in an urn labelled ${1,2,\dots,6}$. Two balls are drawn without replacement. Let $X$ be the larger of the two numbers drawn. Find $\rm{E} [X]$.

Solution Let $X_1, X_2$ denote the draws so that $X=\max(X_1,X_2)$. Then for $k=2,3,4,5,6$ we have \(p(k)={\rm{P}}(X=k)={\rm{P}}(\max(X_1,X_2)=k)={\rm{P}}(\max(X_1,X_2)\leq k)-{\rm{P}}(\max(X_1,X_2)\leq k-1)\)

\[==\frac{\binom{k}{2}}{\binom{6}{2}}-\frac{\binom{k-1}{2}}{\binom{6}{2}} =\frac{k(k-1)-(k-1)(k-2)}{30} =\frac{2k-3}{30}.\]

Hence, \({\rm{E}}[X]=\sum_{k=2}^6 k \frac{2k-3}{30} = \frac{1}{15} \sum_{k=2}^6 k^2 -\frac{1}{10} \sum_{k=2}^6 k =\frac{1}{15}(\frac{(13)(7)(6)}{6}-1) -\frac{1}{10}(\frac{(7)(6)}{2}-1)\)

\[=6-2=4.\]
Question 3:

Let $X$ be a random variable with probability density function \(f(x)=\begin{cases} 1, &0<x<1\\ 0, &\text{otherwise} \end{cases}\) Compute ${\rm{E}}[e^X]$ in two different ways:

(a) By finding the probability density function of $Y=e^X$

(b) By applying the formula \({\rm{E}}[g(X)]=\int g(x) f(x)\,dx\)

Solution

(a) Let $Y=e^X$. We will find cdf of $Y$ first. For $1<y<e$ we have is \({\rm{P}}(Y\leq y)= {\rm{P}}(e^X\leq y)={\rm{P}}(X\leq \ln y) = \int_0^{\ln y} 1 dx=\ln y.\) By differentiating $F_Y$, we obtain the density function of $Y$ as \(f_Y(y)=\frac{1}{y}\, \hskip 2pt 1\leq y\leq e.\) Hence, \({\rm{E}}[Y]=\int_1^e y \frac{1}{y}\,dy=e-1.\) (b) Using the formula directly, we have \({\rm{E}}[e^X]=\int_{-\infty}^{\infty} e^x f(x)dx=\int_{0}^{1} e^x dx=e-1.\)

Recitation 6 week of Oct 10, 2022

Midterm 1 Review

Recitation 5 week of Oct 3, 2022

Question 1:

Suppose that the length of a phone call in minutes is a random variable with pdf

\[f(x)=\begin{cases}\frac{1}{10}e^{-\frac{x}{10}}, &\text{ if }x\geq 0,\\ 0, &\text{ if }x<0. \end{cases}\]

If someone arrives immediately ahead of you at a public telephone booth, find the probability that you will have to wait

(a) more than 10 minutes

(b) between 10 and 20 minutes.

Solution

(a)

\[\rmP(X>10)=\int_{10}^{\infty} \frac{1}{10}e^{-\frac{x}{10}}\,dx=e^{-1}\approx 0.368\]

(b)

\[\rmP(10<X<20)=\int_{10}^{20} \frac{1}{10}e^{-\frac{x}{10}}\,dx=e^{-1}-e^{-2}\approx 0.233\]
Question 2:

Let

\[f(x)=\begin{cases}e^{-x}, &\text{ if }x\geq 0,\\ 0, &\text{ if }x<0. \end{cases}\]

(a) Show that $f$ is a pdf.

(b) $X$ be a random variable with pdf $f$. Show that

\[\rmP(X>s+t \,\big| X>t)=\rmP(X>s)\]

for all $s,t \geq 0$.

Solution

(a) Clearly $f$ is nonnegative and

\(\int_{-\infty}^{\infty}f(x)\,dx=\int_0^{\infty} e^{-x}\,dx=1.\) So, $f$ is a pdf.

(b) On one hand, \(\rmP(X>s+t\big| X>t)=\frac{ \rmP(X>s+t, X>t)}{\rmP(X>t)}=\frac{ \rmP(X>s+t)}{\rmP(X>t)}=\frac{\int_{s+t}^{\infty} e^{-x}\,dx }{\int_{t}^{\infty} e^{-x}\,dx}=\frac{e^{-(s+t)}}{e^{-t}}=e^{-s}\) and on the other hand \(\rmP(X>s)={\int_{s}^{\infty} e^{-x}\,dx}=e^{-s}.\)

Hence the identity follows.

Question 3:

Let $X$ be a discrete random variable with probability mass function

\[p(1)=\frac{1}{4}, \hskip 5pt p(2)=\frac{1}{2}, \hskip 5pt p(3)=\frac{1}{8}, \hskip 5pt p(4)=\frac{1}{8}.\]

Find the cumulative distribution function $F$ of $X$.

Solution

Now for $s<1$, we have

\[F(s)=\rmP(X\leq s)=0.\]

For $1\leq s<2$, we have

\[F(s)=\rmP(X\leq s)=\rmP(X=1)=p(1)=\frac{1}{4}.\]

For $2\leq s<3$, we have

\[F(s)=\rmP(X\leq s)=\rmP(X=1)+\rmP(X=2)=p(1)+p(2)=\frac{1}{4}+\frac{1}{2}=\frac{3}{4}.\]

For $3\leq s<4$, we have

\[F(s)=\rmP(X\leq s)=\rmP(X=1)+\rmP(X=2)+\rmP(X=3)=p(1)+p(2)+p(3)=\frac{1}{4}+\frac{1}{2}+\frac{1}{8}=\frac{7}{8}.\]

Finally, for $s\geq 4$, we have

\[F(s)=\rmP(X\leq s)=\rmP(X=1)+\rmP(X=2)+\rmP(X=3)+\rmP(X=4)=p(1)+p(2)+p(3)+p(4)=\frac{1}{4}+\frac{1}{2}+\frac{1}{8}+\frac{1}{8}=1\]

Collecting this information, we can write

\[F(s)=\begin{cases} 0, &s<1\\ \frac{1}{4}, & 1\leq s<2\\ \frac{3}{4}, & 2\leq s<3\\ \frac{7}{8}, & 3\leq s<3\\ 1, &s\geq 4. \end{cases}\]

Recitation 4 week of Sep 26, 2022

Question 1:

Suppose we toss 2 fair dice. Let $A_1$ be the event that the sum of the dice is 6, $A_2$ be the event that the sum of two dice is 7, $B$ be the event that the first die equals 4 and $C$ is the event that the second die is 3.

(a) Are $A_1$ and $B$ independent?

(b) Are $A_2$ and $B$ independent?

(c) Are $A_2$, $B$, $C$ are pairwise independent?

(d) Are $A_2$, $B$, $C$ are independent?

Solution

Note that $#\Omega=6^2=36$and

\[A_1=\{(1,5),(2,4),(3,3),(4,2),(5,1)\},\] \[A_2=\{(1,6),(2,5),(3,4),(4,3),(5,2),(6,1)\},\] \[B=\{(4,1),(4,2),(4,3),(4,4),(4,5),(4,6)\},\] \[C=\{(1,3),(2,3),(3,3),(4,3),(5,3),(6,3)\},\] \[A_1\cap B=\{(4,2)\},\] \[A_2\cap B=\{(4,3)\},\] \[A_2\cap C= \{(4,3)\},\] \[B\cap C= \{(4,3)\},\] \[A_2\cap B\cap C=\{(4,3)\}.\]

(a) $A_1$ and $B$ are not independent since

\[\rmP(A_1\cap B)=\frac{1}{36}\neq \frac{5}{36}\frac{6}{36}=\rmP(A_1)\rmP(B).\]

(b) $A_2$ and $B$ are independent since

\[\rmP(A_2\cap B)=\frac{1}{36}= \frac{6}{36}\frac{6}{36}=\rmP(A_2)\rmP(B).\]

(c) $A_2, B,C$ are pairwise independent since

\(\rmP(A_2\cap B)=\frac{1}{36}= \frac{6}{36}\frac{6}{36}=\rmP(A_2)\rmP(B)\)
\(\rmP(A_2\cap C)=\frac{1}{36}=\frac{6}{36}\frac{6}{36}=\rmP(A_2)\rmP(C)\)
\(\rmP(B\cap C)=\frac{1}{36}= \frac{6}{36}\frac{6}{36}=\rmP(B)\rmP(C) .\)

(d) $A_2, B,C$ are \underline{not} independent although they are pairwise independent since

\[\rmP(A_2\cap B \cap C)=\frac{1}{36}\neq \frac{6}{36}\frac{6}{36} \frac{6}{36}=\rmP(A_2)\rmP(B) \rmP(C).\]
Question 2:

An infinite sequence of independent trials is to be performed. Each trial result in success with probability $p$ and a failure with probability $1-p$. What is the probability that

(a) at least 1 success occurs in the first $n$ trials?

(b) exactly $k$ successes occurs in the first $n$ trials?

Solution

(a) It is easier to compute the probability of the complement, that is of no success in the first $n$ trials. If we let $A_i$ denote the event of a failure in the $i$th trial, then by independence we have

\(\rmP(A_1\cap A_2\dots A_n)=\rmP(A_1)\rmP(A_2)\dots \rmP(A_n)=(1-p)^n.\) Hence \(\rmP(\text{at least 1 success})=1-(1-p)^n.\)

(b) Any particular sequence of the n trials with $k$ successes and $(n-k)$ failures will occur with probability $p^k(1-p)^{n-k}$. As there are $\binom{n}{k}$ such sequences, we have

\(\rmP(\text{exactly }k \text{ successes})=\binom{n}{k} p^k(1-p)^{n-k}.\)

Question 3:

Suppose $\rmP(A)=0.3$ and $\rmP(B)=0.6$.

(a) If $A$ and $B$ are disjoint, what is $\rmP(A\cup B)$?

(b) If $A$ and $B$ are independent, what is $\rmP(A\cup B)$?

Solution

(a) If $A$ and $B$ are disjoint, then \(\rmP(A\cup B)=\rmP(A)+\rmP(B)=0.3+0.6=0.9.\)

(b) If $A$ and $B$ are independent, then \(\rmP(A\cup B)=\rmP(A)+\rmP(B)-\rmP(A\cap B)=\rmP(A)+\rmP(B)-\rmP(A)\rmP(B)=0.3+0.6-0.18=0.72.\)

Recitation 3 week of Sep 19, 2022

Question 1:

An urn has $n-3$ green balls and $3$ red balls. There are $l$ draws with replacement. Let

\[B = \{\text{at least one red ball is drawn}\}.\]

Find $P(B)$ using inclusion/exclusion principle.

Solution

Let \(B_k = \{k\text{th ball drawn is red}\}.\) Then \(\rm{P}(B)=\rm{P}\left(\bigcup_{k=1}^l B_k \right).\) Using inclusion/exclusion principle, we have \({\rm{P}}(B)={\rm{P}}\left(\bigcup_{k=1}^l B_k \right)=\sum_{k=1}^l(-1)^{k+1}\sum_{1\leq i_1<\dots <i_k \leq l} {\rm{P}}(B_{i_1}\cap \dots \cap B_{i_k}).\)Note that \({\rm{P}}(B_{i_1}\cap \dots \cap B_{i_k})=\frac{3^k}{n^k}.\)

Hence 
\[{\rm{P}}(B)={\rm{P}}\left(\bigcup_{k=1}^l B_k \right)=\sum_{k=1}^l(-1)^{k+1}\sum_{1\leq i_1<\dots <i_k \leq l} \frac{3^k}{n^k}\] \[=\sum_{k=1}^l(-1)^{k+1}\frac{3^k}{n^k}\#\{(i_1,\dots,i_k):1\leq i_1<\dots <i_k \leq l\} =\sum_{k=1}^l(-1)^{k+1} \binom{l}{k} \frac{3^k}{n^k}\] \[=1-(1-\frac{3}{n})^l=1-\left(\frac{n-3}{n}\right)^l\]

Note we can also compute this probability without inclusion/exclusion:

\[{\rm{P}}(B)=1- {\rm{P}}(B^c)=1-\left(\frac{n-3}{n}\right)^l.\]
Question 2:

Two fair dice are rolled. Let $X$ be the maximum of the two numbers.

(a) Find the possible values of $X$.

(b) Find the probabilities $P(X\leq k)$ for all integers $k$.

(c) Find the probability mass function of $X$.

Hint: Noticing that \(P(X = k) = P(X \leq k) - P(X \leq k-1)\) can save you some work.

Solution

(a) $X$ can take values $1,2,3,4,5,6$.

(b) Note

\[{\rm{P}}(X\leq k) = {\rm{P}}((i,j):X(i,j)\leq k)= {\rm{P}}((i,j):\max\{i,j\}\leq k)={\rm{P}}((i,j):i\leq k, j\leq k)=\frac{k^2}{36}\]

(c) Using the hint $P(X = k) = P(X \leq k) - P(X \leq k-1)$, we have

\(P(X = k) = \frac{k^2}{36}-\frac{(k-1)^2}{36}=\frac{2k-1}{36}.\)

Question 3:

There are 99 fair coins and one coin which has tail on both sides. Jim chooses a coin randomly, tossed it 5 times, and discovered that he got all 5 tails. What is the probability that Jim picked the unfair coin? (Assuming that all coins are equally likely to be chosen)

Solution

Let $A={\text{ Jim picks the unfair coin}}$ and $B={\text{he gets 5 tails in 5 tosses}}$. Note that \({\rm{P}}(A\cap B) =\frac{1}{100}\) and ${\rm{P}}(B)={\rm{P}}(\text{choosing a fair coin})\frac{1}{2^5}+{\rm{P}}(\text{choosing the unfair coin}) 1= \frac{1}{100}1+\frac{99}{100}\frac{1}{2^5}$. Then using the conditional probabilities, we see \({\rm{P}}(A\big|B)=\frac{{\rm{P}} (A\cap B)}{{\rm{P} (B)}}=\frac{\frac{1}{100}}{\frac{1}{100}1+\frac{99}{100}\frac{1}{2^5}}.\)

Recitation 2 week of Sep 12, 2022

Question 1:

If three balls are randomly drawn from a ball containing 6 white and 5 black balls, what is the probability that one of the thrown balls is white and the other two black?

Solution

First number the balls from \(\{1,2,\dots,11\}\), where \(\{1,2,\dots,6\}\) corresponds to white balls and \(\{7,8,\dots,11\}\) to black balls. We pick 3 numbers and want to know the probability that 2 of them are from \(\{7,8,\dots,11\}\) and one of them is from \(\{1,2,\dots,6\}\).

Solution with order:

\(\Omega=\{(s_1,s_2,s_3):s_i\neq s_j \text{ for all }i\neq j\}\). Then \(\#\Omega=(11)_3=11\cdot 10\cdot 9\). Let \(A:=\{ (s_1,s_2,s_3) \in \Omega: \text{two of them} \in \{7,\dots, 11\} \text{ and } \text{ one of them } \in \{1,\dots, 6\}\}.\) Then \(\#A= 3 \cdot (5\cdot 4 \cdot 6)\) and hence \(\rmP(A)= \frac{3\cdot5\cdot 4\cdot 6}{11\cdot 10\cdot 9 } =\frac{4}{11}.\)

Solution without order:

Let \(\tilde{\Omega}=\{B\subset \{1,2,\dots,11\}: \#B=3\}\). Then \(\#\tilde{\Omega}=\binom{11}{3}\). Let \(\tilde{A}\in \tilde{\Omega}\) be such that

\[\tilde{A}:=\{\{s_1,s_2,s_3\}\in \tilde{\Omega}: s_1 \in \{1,\dots, 6\}, s_2,s_3\in \{7,\dots,11\}\}.\]

Then \(\#\tilde{A}=\binom{6}{1} \cdot \binom{5}{2}\) and hence

\[\rmP(\tilde{A})=\frac{\binom{6}{1}\cdot \binom{5}{2}}{\binom{11}{3}}=\frac{4}{11}.\]
Question 2:

In a game of bridge the entire deck of 52 cards is dealt out to 4 players.

(a) What is the probability that one of the players receives all 13 spades?

Solution

There are \(\binom{52}{13}\cdot \binom{39}{13}\cdot \binom{26}{13} \cdot \binom{13}{13}\) possible ways to divide the cards among 4 people. There are 4 ways to choose who gets all the spades and there are \(\binom{39}{13}\cdot \binom{26}{13} \cdot \binom{13}{13}\) possible ways to divide the rest of the cards among the rest 3 people. Hence

\[\rmP(A)=\frac{4\cdot \binom{39}{13}\cdot \binom{26}{13} \cdot \binom{13}{13} }{\binom{52}{13}\cdot \binom{39}{13}\cdot \binom{26}{13} \cdot \binom{13}{13}} =\frac{4 }{\binom{52}{13}} \approx 6 \times 10^{-12}.\]

(b) What is the probability that each player receives one ace?

Solution

There are \(\binom{52}{13}\cdot \binom{39}{13}\cdot \binom{26}{13} \cdot \binom{13}{13}\) possible ways to divide the cards among 4 people. If we put aside the aces, there are \(\binom{48}{12}\cdot \binom{36}{12} \cdot \binom{24}{12} \cdot \binom{12}{12}\) possible ways to divide the rest of the cards among the rest 4 people. As there are \(4!\) ways of dividing the 4 aces, the desired probability is \(\rmP(A)=\frac{4!\cdot \binom{48}{12}\cdot \binom{36}{12} \cdot \binom{24}{12} \cdot \binom{12}{12} }{\binom{52}{13}\cdot \binom{39}{13}\cdot \binom{26}{13} \cdot \binom{13}{13}} \approx 0.1\)

Question 3:

(a) If $P (E) = 0.9$ and $P (F ) = 0.8$, show that

(b) In general, prove Bonferroni’s inequality: \(P (E \cap F ) \geq P (E) + P (F ) - 1.\)

Solution

(a) Using inclusion/exclusion principle, we have \(\rmP(E\cap F)=\rmP(E)+\rmP(F)-\rmP(E\cup F)= 0.9+0.8 - \rmP(E\cup F)\geq 1.7-1=0.7.\)

(b) Similarly using inclusion/exclusion principle, we have \(\rmP(E\cap F)=\rmP(E)\rmP(F)-\rmP(E\cup F)\geq \rmP(E)\rmP(F) - 1.\)

Recitation 1 week of Sep 5, 2022

Question 1:

Suppose there are 7 identical oranges and 5 distinct bins.

Find the number of ways to distribute the oranges in the bins.

Solution

Let us make sequences of \(o\)’s and $$\big \('s any such sequence that has 4\)\big \('s and 7\)o$$’s represents a way of arranging balls into boxes.E.g.
\[\, o\,o \, \big| \, o\,o \, \big| \, o \, \big| \, o \, \big| \, o \,\]

would represent 2 oranges in first bin, 2 oranges in the second bin and 1 orange in each of the third, fourth and fifth bin. Altogether, there are $7+5-1$ symbols. We have

\(=\binom{7+5-1}{7}=\binom{11}{7}\) many ways to put 7 oranges into 11 spots.

Question 2:

In poker each player gets a hand of 5 cards.

(a) A full house consists of three cards of a value and two cards of another value. How many different full houses are there?

(b) How many hands consist of \(5\) different values?

(c) How many hands has \(2\) distinct pairs?

Solution (a)

\[\underbrace{ \binom{13}{1} }_{ \text{rank of the triple}}\times \underbrace{\binom{4}{3}}_{ \text{suits of the triple}} \times \underbrace{\binom{12}{1}}_{{ \text{rank of the pair}}} \times \underbrace{\binom{4}{2}}_{{ \text{suits of the pair}}}\]

(b) We can choose \(\binom{13}{5}\) ways the ranks of each card and choose \(\binom{4}{1}\) ways the suits of each card. Hence

\[\binom{13}{5} \binom{4}{1}^5\]

(c) We can choose \(\binom{13}{2}\) ways the ranks of the pairs and choose \(\binom{4}{2}\) ways the suits of each pair and we can choose the last card in \(\binom{44}{1}\) ways. Hence

\(\binom{13}{2} \binom{4}{2}^2 \binom{44}{1}\)